Menu Close

prove-that-1-x-1-1-x-4-




Question Number 80039 by jagoll last updated on 30/Jan/20
prove that  (1+x)(1+(1/x))≥4
$${prove}\:{that} \\ $$$$\left(\mathrm{1}+{x}\right)\left(\mathrm{1}+\frac{\mathrm{1}}{{x}}\right)\geqslant\mathrm{4} \\ $$
Commented by mr W last updated on 30/Jan/20
only for x∈R^+   1+x≥2(√x)  1+(1/x)≥2(1/( (√x)))  (1+x)(1+(1/x))≥2(√x)×2(1/( (√x)))=4  or  (1+x)(1+(1/x))=2+x+(1/x)≥2+2=4    for x∈R^−   (1+x)(1+(1/x))=2+x+(1/x)=2−(−x+(1/(−x)))≤2−2=0
$${only}\:{for}\:{x}\in{R}^{+} \\ $$$$\mathrm{1}+{x}\geqslant\mathrm{2}\sqrt{{x}} \\ $$$$\mathrm{1}+\frac{\mathrm{1}}{{x}}\geqslant\mathrm{2}\frac{\mathrm{1}}{\:\sqrt{{x}}} \\ $$$$\left(\mathrm{1}+{x}\right)\left(\mathrm{1}+\frac{\mathrm{1}}{{x}}\right)\geqslant\mathrm{2}\sqrt{{x}}×\mathrm{2}\frac{\mathrm{1}}{\:\sqrt{{x}}}=\mathrm{4} \\ $$$${or} \\ $$$$\left(\mathrm{1}+{x}\right)\left(\mathrm{1}+\frac{\mathrm{1}}{{x}}\right)=\mathrm{2}+{x}+\frac{\mathrm{1}}{{x}}\geqslant\mathrm{2}+\mathrm{2}=\mathrm{4} \\ $$$$ \\ $$$${for}\:{x}\in{R}^{−} \\ $$$$\left(\mathrm{1}+{x}\right)\left(\mathrm{1}+\frac{\mathrm{1}}{{x}}\right)=\mathrm{2}+{x}+\frac{\mathrm{1}}{{x}}=\mathrm{2}−\left(−{x}+\frac{\mathrm{1}}{−{x}}\right)\leqslant\mathrm{2}−\mathrm{2}=\mathrm{0} \\ $$
Commented by Tony Lin last updated on 30/Jan/20
[1^2 +((√x))^2 ][1^2 +((1/( (√x))))^2 ]≥(1+1)^2 =4
$$\left[\mathrm{1}^{\mathrm{2}} +\left(\sqrt{{x}}\right)^{\mathrm{2}} \right]\left[\mathrm{1}^{\mathrm{2}} +\left(\frac{\mathrm{1}}{\:\sqrt{{x}}}\right)^{\mathrm{2}} \right]\geqslant\left(\mathrm{1}+\mathrm{1}\right)^{\mathrm{2}} =\mathrm{4} \\ $$
Commented by MJS last updated on 30/Jan/20
y=(1+x)(1+(1/x))  defined for x∈R\{0}  (1+x)(1+(1/x))<4  x+(1/x)<2  (1) x>0  x^2 −2x+1<0  (x−1)^2 <0 wrong ⇒ (1+x)(1+(1/x))≥4  (2) x<0  x^2 −2x+1>0  (x−1)^2 >0 true ∀x<0 ⇒ (1+x)(1+(1/x))<4   { (((1+x)(1+(1/x))≥4; x>0)),(((1+x)(1+(1/x))<4; x<0)) :}
$${y}=\left(\mathrm{1}+{x}\right)\left(\mathrm{1}+\frac{\mathrm{1}}{{x}}\right) \\ $$$$\mathrm{defined}\:\mathrm{for}\:{x}\in\mathbb{R}\backslash\left\{\mathrm{0}\right\} \\ $$$$\left(\mathrm{1}+{x}\right)\left(\mathrm{1}+\frac{\mathrm{1}}{{x}}\right)<\mathrm{4} \\ $$$${x}+\frac{\mathrm{1}}{{x}}<\mathrm{2} \\ $$$$\left(\mathrm{1}\right)\:{x}>\mathrm{0} \\ $$$${x}^{\mathrm{2}} −\mathrm{2}{x}+\mathrm{1}<\mathrm{0} \\ $$$$\left({x}−\mathrm{1}\right)^{\mathrm{2}} <\mathrm{0}\:\mathrm{wrong}\:\Rightarrow\:\left(\mathrm{1}+{x}\right)\left(\mathrm{1}+\frac{\mathrm{1}}{{x}}\right)\geqslant\mathrm{4} \\ $$$$\left(\mathrm{2}\right)\:{x}<\mathrm{0} \\ $$$${x}^{\mathrm{2}} −\mathrm{2}{x}+\mathrm{1}>\mathrm{0} \\ $$$$\left({x}−\mathrm{1}\right)^{\mathrm{2}} >\mathrm{0}\:\mathrm{true}\:\forall{x}<\mathrm{0}\:\Rightarrow\:\left(\mathrm{1}+{x}\right)\left(\mathrm{1}+\frac{\mathrm{1}}{{x}}\right)<\mathrm{4} \\ $$$$\begin{cases}{\left(\mathrm{1}+{x}\right)\left(\mathrm{1}+\frac{\mathrm{1}}{{x}}\right)\geqslant\mathrm{4};\:{x}>\mathrm{0}}\\{\left(\mathrm{1}+{x}\right)\left(\mathrm{1}+\frac{\mathrm{1}}{{x}}\right)<\mathrm{4};\:{x}<\mathrm{0}}\end{cases} \\ $$
Commented by mr W last updated on 30/Jan/20
typo sir?  (1+x)(1+(1/x))<4; x<0  should be:  (1+x)(1+(1/x))≤0; x<0
$${typo}\:{sir}? \\ $$$$\left(\mathrm{1}+{x}\right)\left(\mathrm{1}+\frac{\mathrm{1}}{{x}}\right)<\mathrm{4};\:{x}<\mathrm{0} \\ $$$${should}\:{be}: \\ $$$$\left(\mathrm{1}+{x}\right)\left(\mathrm{1}+\frac{\mathrm{1}}{{x}}\right)\leqslant\mathrm{0};\:{x}<\mathrm{0} \\ $$
Commented by MJS last updated on 30/Jan/20
You are right but with my method I only  examined if it′s <4.
$$\mathrm{You}\:\mathrm{are}\:\mathrm{right}\:\mathrm{but}\:\mathrm{with}\:\mathrm{my}\:\mathrm{method}\:\mathrm{I}\:\mathrm{only} \\ $$$$\mathrm{examined}\:\mathrm{if}\:\mathrm{it}'\mathrm{s}\:<\mathrm{4}. \\ $$
Commented by jagoll last updated on 31/Jan/20
thank you
$${thank}\:{you}\: \\ $$
Answered by $@ty@m123 last updated on 31/Jan/20
Let y=(1+x)(1+(1/x))  ⇒y=2+x+(1/x)  (dy/dx)=1−(1/x^2 )  For max. or min.  (dy/dx)=0  1−(1/x^2 )=0  ⇒x=±1  y∣_(x=1)  =4,  y∣_(x=−1) =0  (d^2 y/dx^2 )=(1/x^3 )  [(d^2 y/dx^2 )]_(x=1) =1>0  ∴ y is mininmum at x=1  and min.(y)=4
$${Let}\:{y}=\left(\mathrm{1}+{x}\right)\left(\mathrm{1}+\frac{\mathrm{1}}{{x}}\right) \\ $$$$\Rightarrow{y}=\mathrm{2}+{x}+\frac{\mathrm{1}}{{x}} \\ $$$$\frac{{dy}}{{dx}}=\mathrm{1}−\frac{\mathrm{1}}{{x}^{\mathrm{2}} } \\ $$$${For}\:{max}.\:{or}\:{min}. \\ $$$$\frac{{dy}}{{dx}}=\mathrm{0} \\ $$$$\mathrm{1}−\frac{\mathrm{1}}{{x}^{\mathrm{2}} }=\mathrm{0} \\ $$$$\Rightarrow{x}=\pm\mathrm{1} \\ $$$${y}\mid_{{x}=\mathrm{1}} \:=\mathrm{4}, \\ $$$${y}\mid_{{x}=−\mathrm{1}} =\mathrm{0} \\ $$$$\frac{{d}^{\mathrm{2}} {y}}{{dx}^{\mathrm{2}} }=\frac{\mathrm{1}}{{x}^{\mathrm{3}} } \\ $$$$\left[\frac{{d}^{\mathrm{2}} {y}}{{dx}^{\mathrm{2}} }\right]_{{x}=\mathrm{1}} =\mathrm{1}>\mathrm{0} \\ $$$$\therefore\:{y}\:{is}\:{mininmum}\:{at}\:{x}=\mathrm{1} \\ $$$${and}\:{min}.\left({y}\right)=\mathrm{4} \\ $$

Leave a Reply

Your email address will not be published. Required fields are marked *